Isomorphism: matrix determinant

Click For Summary
The discussion revolves around determining if the map \varphi, defined as the determinant of a matrix A, is an isomorphism between the structures <M2(R), usual multiplication> and <R, usual multiplication>. The determinant is expressed as ad-bc, and the user is attempting to prove that \varphi is one-to-one, onto, and preserves operations. They express difficulty in showing that the function preserves operations, despite being able to establish it as one-to-one and onto. The conversation clarifies that the question is not asserting that it is an isomorphism but rather asking whether it can be proven as such. Ultimately, the user acknowledges a misunderstanding regarding the nature of the question.
kala
Messages
21
Reaction score
0
Determine whether the given map \varphi is an isomorphism of the first binary structure with the second.
< M2(R ), usual multiplication > with <R, usual multiplication> where \varphi(A) is the determinant of matrix A.

The determinant of the matrix is ad-bc, so \varphi(A)=ad-bc.
For this to be an isomorphism, I have to show that the function is one to one, onto and preserves the operations.
I'm having trouble getting this to work. Any suggestions?
 
Physics news on Phys.org
kala said:
I'm having trouble getting this to work.
Maybe it can't...
 
According to the book it is suppose to be an isomorphism, the question says it is. I can get it to be one to one and onto, but i am having trouble with it preserving the operations.
 
You just said that you must prove the function is "one to one". That is that two different matrices, such as
\begin{bmatrix}2 &amp; 1 \\ 1 &amp; 1\end{bmatrix}
and
\begin{bmatrix}3 &amp; 2\\ 1 &amp; 1\end{bmatrix}
must not have the same determinant. Is that true?

I think you should to reread that problem.
 
kala said:
the question says it is.
No it doesn't. The question is asking you whether or not it is.
 
Oh duh... That was stupid... It doesn't have to be. Thanks
 
Question: A clock's minute hand has length 4 and its hour hand has length 3. What is the distance between the tips at the moment when it is increasing most rapidly?(Putnam Exam Question) Answer: Making assumption that both the hands moves at constant angular velocities, the answer is ## \sqrt{7} .## But don't you think this assumption is somewhat doubtful and wrong?

Similar threads

  • · Replies 4 ·
Replies
4
Views
1K
Replies
2
Views
2K
  • · Replies 8 ·
Replies
8
Views
2K
  • · Replies 1 ·
Replies
1
Views
3K
  • · Replies 4 ·
Replies
4
Views
1K
  • · Replies 4 ·
Replies
4
Views
2K
  • · Replies 6 ·
Replies
6
Views
2K
  • · Replies 1 ·
Replies
1
Views
3K
  • · Replies 45 ·
2
Replies
45
Views
6K
  • · Replies 1 ·
Replies
1
Views
2K